Difference between revisions of "1988 AJHSME Problems/Problem 5"

m
m (Solution)
 
(One intermediate revision by one other user not shown)
Line 22: Line 22:
 
<math>\text{(A)}\ 20^\circ \qquad \text{(B)}\ 40^\circ \qquad \text{(C)}\ 50^\circ \qquad \text{(D)}\ 70^\circ \qquad \text{(E)}\ 120^\circ</math>
 
<math>\text{(A)}\ 20^\circ \qquad \text{(B)}\ 40^\circ \qquad \text{(C)}\ 50^\circ \qquad \text{(D)}\ 70^\circ \qquad \text{(E)}\ 120^\circ</math>
 
==Solution==
 
==Solution==
We have that <math>20^{\circ}+\angle ABD +\angle CBD=160^{\circ}</math>, or <math>\angle ABD +\angle CBD=140^{\circ}</math>. Since <math>\angle CBD</math> is a right angle, we have <math>\angle ABD=140^{\circ}-90^{\circ}=50^{\circ}\Rightarrow \mathrm{(C)}</math>.
+
We have that <math>20^{\circ}+\angle ABD +\angle CBD=160^{\circ}</math>, or <math>\angle ABD +\angle CBD=140^{\circ}</math>. Since <math>\angle CBD</math> is a right angle, we have <math>\angle ABD=140^{\circ}-90^{\circ}=50^{\circ}\Rightarrow \boxed{\mathrm{(C)}}</math>.
  
 
==See Also==
 
==See Also==
Line 28: Line 28:
 
{{AJHSME box|year=1988|num-b=4|num-a=6}}
 
{{AJHSME box|year=1988|num-b=4|num-a=6}}
 
[[Category:Introductory Geometry Problems]]
 
[[Category:Introductory Geometry Problems]]
 +
{{MAA Notice}}

Latest revision as of 16:53, 17 December 2020

Problem

If $\angle \text{CBD}$ is a right angle, then this protractor indicates that the measure of $\angle \text{ABC}$ is approximately

[asy] unitsize(36); pair A,B,C,D; A=3*dir(160); B=origin; C=3*dir(110); D=3*dir(20); draw((1.5,0)..(0,1.5)..(-1.5,0)); draw((2.5,0)..(0,2.5)..(-2.5,0)--cycle); draw(A--B); draw(C--B); draw(D--B); label("O",(-2.5,0),W); label("A",A,W); label("B",B,S); label("C",C,W); label("D",D,E); label("0",(-1.8,0),W); label("20",(-1.7,.5),NW); label("160",(1.6,.5),NE); label("180",(1.7,0),E); [/asy]

$\text{(A)}\ 20^\circ \qquad \text{(B)}\ 40^\circ \qquad \text{(C)}\ 50^\circ \qquad \text{(D)}\ 70^\circ \qquad \text{(E)}\ 120^\circ$

Solution

We have that $20^{\circ}+\angle ABD +\angle CBD=160^{\circ}$, or $\angle ABD +\angle CBD=140^{\circ}$. Since $\angle CBD$ is a right angle, we have $\angle ABD=140^{\circ}-90^{\circ}=50^{\circ}\Rightarrow \boxed{\mathrm{(C)}}$.

See Also

1988 AJHSME (ProblemsAnswer KeyResources)
Preceded by
Problem 4
Followed by
Problem 6
1 2 3 4 5 6 7 8 9 10 11 12 13 14 15 16 17 18 19 20 21 22 23 24 25
All AJHSME/AMC 8 Problems and Solutions

The problems on this page are copyrighted by the Mathematical Association of America's American Mathematics Competitions. AMC logo.png